Wealth Distribution- Productivity: Weaken

This topic has expert replies
User avatar
Master | Next Rank: 500 Posts
Posts: 229
Joined: Tue Jan 13, 2009 6:56 am
Thanked: 8 times
GMAT Score:700

Wealth Distribution- Productivity: Weaken

by Uri » Wed Feb 25, 2009 8:44 am
Contrary to the statements of labor leaders, the central economic problem facing America today is not the distribution of wealth. It is productivity. With the productivity of U.S. industry stagnant, or even declining slightly, the economic pie is no longer growing. Labor leaders, of course, point to what they consider an unfair distribution of the slices of pie to justify their demands for further increases in wages and benefits. And in the past, when the pie was still growing, management could afford to acquiesce. No longer. Until productivity resumes its growth, there can be no justification for further increases in the compensation of workers.
Which of the following statements by a labor leader focuses on the logical weakness in the argument above?
(A) Although the economic pie is no longer growing, the portion of the pie allocated to American workers remains unjustly small.
(B) If management fails to accommodate the demands of workers, labor leaders will be forced to call strikes that will cripple the operation of industry.
(C) Although productivity is stagnant, the U.S. population is growing, so that the absolute size of the economic pie continues to grow as well.
(D) As a labor leader, I can be concerned only with the needs of working people, not with the problems faced by management.
(E) The stagnation of U.S. industry has been caused largely by factors—such as foreign competition—beyond the control of American workers.
Please justify your answer with reasoning.
OA: [spoiler](A)[/spoiler]

Junior | Next Rank: 30 Posts
Posts: 25
Joined: Mon Feb 09, 2009 11:11 am

by venusgarg » Wed Feb 25, 2009 7:58 pm
IMO A

Master | Next Rank: 500 Posts
Posts: 239
Joined: Wed Feb 11, 2009 2:50 am

by delhiboy1979 » Thu Feb 26, 2009 2:55 am
Hi Uri,

Ill try to reason the answers:
Which of the following statements by a labor leader focuses on the logical weakness in the argument above?

(A) Although the economic pie is no longer growing, the portion of the pie allocated to American workers remains unjustly small.
*the issue that the labor leaders are trying to address is the imbalance in wages between low paid workers and managers, so even though the pie is no longer growing, the share is still the same as when it was.

(B) If management fails to accommodate the demands of workers, labor leaders will be forced to call strikes that will cripple the operation of industry.
* strikes! its more of a threat by the labour leaders than a weakness in the argument above

(C) Although productivity is stagnant, the U.S. population is growing, so that the absolute size of the economic pie continues to grow as well.
*population? this has absolutely no relation with the lower paid workers

(D) As a labor leader, I can be concerned only with the needs of working people, not with the problems faced by management.
* again irrlevant, does not address the labour leaders intital argument

(E) The stagnation of U.S. industry has been caused largely by factors—such as foreign competition—beyond the control of American workers.
*out of scope, no mention of foreign competetion

HTH

User avatar
Master | Next Rank: 500 Posts
Posts: 229
Joined: Tue Jan 13, 2009 6:56 am
Thanked: 8 times
GMAT Score:700

by Uri » Thu Feb 26, 2009 1:00 pm
delhiboy1979 wrote:Hi Uri,

Ill try to reason the answers:
Which of the following statements by a labor leader focuses on the logical weakness in the argument above?

(A) Although the economic pie is no longer growing, the portion of the pie allocated to American workers remains unjustly small.
*the issue that the labor leaders are trying to address is the imbalance in wages between low paid workers and managers, so even though the pie is no longer growing, the share is still the same as when it was.

(B) If management fails to accommodate the demands of workers, labor leaders will be forced to call strikes that will cripple the operation of industry.
* strikes! its more of a threat by the labour leaders than a weakness in the argument above

(C) Although productivity is stagnant, the U.S. population is growing, so that the absolute size of the economic pie continues to grow as well.
*population? this has absolutely no relation with the lower paid workers

(D) As a labor leader, I can be concerned only with the needs of working people, not with the problems faced by management.
* again irrlevant, does not address the labour leaders intital argument

(E) The stagnation of U.S. industry has been caused largely by factors—such as foreign competition—beyond the control of American workers.
*out of scope, no mention of foreign competetion

HTH
Thanks for your answer.

But I still have two doubts. First, as (C) says, if the absolute value of the economic pie continues to grow, then what can be the reason for not raising the salary of the low-paid labourers? Please note that the argument prohibits any further increase in pay of the low-paid labourers on the basis of stagnant growth.

Second, in "weaken" question type, we can bring new information. So, we can not eliminate (E) by simply saying it is "out of scope" or that it has not been mentioned in the argument.

Although I don't find (E) to be a good contender, I am definitely missing some point while considering (C) or perhaps I am assuming too much. Can anyone provide some more justification why we should eliminate (C)?

User avatar
Master | Next Rank: 500 Posts
Posts: 229
Joined: Tue Jan 13, 2009 6:56 am
Thanked: 8 times
GMAT Score:700

by Uri » Thu Feb 26, 2009 1:30 pm
Oh, finally I feel that I have found a good reason to eliminate (C)!

The argument was talking about the distribution of wealth vs productivity. So, the speaker was considering of raising the ratio of payment for management (or other people, perhaps) to workers. Even if absolute economy grows, raising the average payment may not be possible, since absolute payment to workers may also grow. In short, (C) talks about absolute numbers, while our main point was about percentage or ratio. So, (C) should be eliminated.

Please rectify me if I am wrong.

Thanks.

GMAT/MBA Expert

User avatar
GMAT Instructor
Posts: 2228
Joined: Wed Dec 27, 2006 3:28 pm
Location: Montreal, Canada
Thanked: 639 times
Followed by:694 members
GMAT Score:780

by Stacey Koprince » Tue Mar 10, 2009 11:29 am
Received a PM asking me to reply, in particular as to the reasoning for choice C. Sorry it has taken me a while to reply - I was on vacation until yesterday!

The language in this one is not very GMAT-like. I've never seen a GMAT use casual language such as "economic pie" and the complete sentence "No longer." Further, the argument requires you to know what is meant by the term "distribution of wealth" and the real test doesn't expect people to know various economic terms. I wouldn't spend much time on this one.

General flow:
Contrary to <the people who disagree with me>, <something is not true>. It is <actually something else that I believe>. <Followed by lots of stuff about why I believe what I believe...>

Author thinks big problem is NOT distribution of wealth. Labor leaders (LLs) do apparently think this is the case (according to author). Author thinks big problem is productivity.

Productivity is bad, so amount of money moving around isn't getting bigger.
LLs think unfair distrib is problem and demand more $$. When amount of $$ was growing, easier for mgmt to give LLs more $$. But not any more b/c amount of $$ isn't growing. Until more $$, LLs can't justify asking for more.

The question indicates that the answer choices are possible responses by an LL - that is, someone who disagrees with the author. And we're asked to find a weakness - so something an LL would say to weaken the above.

The LLs are concerned with the percentage of the money they (and their people) get. They don't care whether the total is growing or staying the same or shrinking. They think that their portion is inadequate because the money is distributed unevenly.

(A) says that - what you argue, author, about the amount of $$ overall, is totally beside the point. Our point is that we should have a greater share of whatever money there is, period.

(B) out of scope. This could happen (and has in the past) but it doesn't address the particular argument given above.

(C) This is poorly worded. It contradicts what the argument says, although I'm guessing that wasn't the (real) author's intent. The argument says the pie is not growing and this choice says it is growing. Presumably, the intention was that the overall pie is growing, but the per-person share of that pie is not growing because there are more people. But the argument doesn't say that the per-person share isn't growing - the argument just says the "pie" is no longer growing. If this had been made clear, then it might be more clear why this choice is wrong. The statement is supposed to be from an LL who is trying to argue for more money for the laborers. Pointing out that the absolute size is getting bigger doesn't help if the per-person share is not also growing.

(D) out of scope - some LL may feel this way, but that does not address a weakness of the presented argument.

(E) See D.
Please note: I do not use the Private Messaging system! I will not see any PMs that you send to me!!

Stacey Koprince
GMAT Instructor
Director of Online Community
Manhattan GMAT

Contributor to Beat The GMAT!

Learn more about me

Master | Next Rank: 500 Posts
Posts: 116
Joined: Sun Feb 01, 2009 10:56 am

by Musicolo » Tue Mar 10, 2009 2:37 pm
Its clearly A, didnt even have to read the other ones

Master | Next Rank: 500 Posts
Posts: 100
Joined: Tue Aug 05, 2014 3:58 am

by vanessa.m » Sat May 14, 2016 2:42 am
Answer is clearly A